Những câu hỏi liên quan
tth_new
Xem chi tiết
Không Tên
12 tháng 10 2018 lúc 10:56

\(A=\frac{a}{b+c}+\frac{b}{c+a}+\frac{c}{a+b}\)

=>  \(A+3=\frac{a}{b+c}+1+\frac{b}{c+a}+1+\frac{c}{a+b}+1\)

\(=\left(a+b+c\right)\left(\frac{1}{a+b}+\frac{1}{b+c}+\frac{1}{c+a}\right)\)

\(=\frac{1}{2}\left[\left(a+b\right)+\left(b+c\right)+\left(c+a\right)\right]\left(\frac{1}{a+b}+\frac{1}{b+c}+\frac{1}{c+a}\right)\)

\(\ge\frac{1}{2}.3\sqrt[3]{\left(a+b\right)\left(b+c\right)\left(c+a\right)}.3\sqrt[3]{\frac{1}{a+b}.\frac{1}{b+c}.\frac{1}{c+a}}=\frac{9}{2}\)   (AM - GM)

=>  \(A\ge\frac{9}{2}-3=\frac{3}{2}\)  (đpcm)

Bình luận (0)
kudo shinichi
12 tháng 10 2018 lúc 18:45

Đặt \(A=\frac{a}{b+c}+\frac{b}{c+a}+\frac{c}{a+b}\)

\(A=\frac{a^2}{ba+ca}+\frac{b^2}{cb+ba}+\frac{c^2}{ac+bc}\)

Áp dụng BĐT Cauchy-schwarz ta có: 

\(A=\frac{a^2}{ba+ca}+\frac{b^2}{cb+ba}+\frac{c^2}{ac+bc}\ge\frac{\left(a+b+c\right)^2}{2.\left(ab+bc+ca\right)}\)

Ta c/m BĐT phụ \(ab+bc+ca\le\frac{1}{3}.\left(a+b+c\right)^2\)( tự c/m)

Áp dụng: 

\(A\ge\frac{\left(a+b+c\right)^2}{2.\frac{1}{3}\left(a+b+c\right)^2}=\frac{1}{\frac{2}{3}}=\frac{3}{2}\)

                                                 đpcm

Tham khảo nhé~

Bình luận (0)
Ngô Thế Trường ( CRIS DE...
12 tháng 10 2018 lúc 21:40

P = a/(b+c) + b/(c+a) + c/(a+b) 
P + 3 = 1+ a/(b+c) + 1+ b/(c+a) + 1+ c/(a+b) 
P + 3 = (a+b+c)/(b+c) + (a+b+c)/(b+c) + (a+b+c)/(c+a) 
P + 3 = (a+b+c)[1/(b+c) + 1/(c+a) + 1/(a+b)] (*) 

ad bđt cô si cho 3 số: 
2(a+b+c) = (a+b) + (b+c) + (c+a) ≥ 3.³√(a+b)(b+c)(c+a) 
1/(b+c) + 1/(c+a) + 1/(a+b) ≥ 3.³√1/(a+b)(b+c)(c+a) 

nhân lại vế theo vế 2 bđt: 2(a+b+c)[1/(b+c) + 1/(c+a) + 1/(a+b)] ≥ 9 
=> P + 3 ≥ 9/2 => P ≥ 3/2 (đpcm) ; dấu "=" khi a = b = c 
- - - 
cách khác: P = a/(b+c) + b/(c+a) + c/(a+b) 
M = b/(b+c) + c/(c+a) + a/(a+b) 
N = c/(b+c) + a/(c+a) + b/(a+b) 

Thấy: M + N = 3 
P + M = (a+b)/(b+c) + (b+c)/(c+a) + (c+a)/(a+b) ≥ 3 (cô si cho 3 số) 
P + N = (a+c)/(b+c) + (b+a)/(c+a) + (c+b)/(a+b) ≥ 3 (cô si) 

=> 2P + M + N ≥ 6 => 2P + 3 ≥ 6 => P ≥ 3/2 (đpcm) ; đẳng thức khi a = b = c 

Bình luận (0)
nub
Xem chi tiết
Đào Duy Tùng
18 tháng 4 2020 lúc 14:35

mình lớp 5 nên mình ko biếu xui quá

Bình luận (0)
 Khách vãng lai đã xóa
tth_new
31 tháng 5 2020 lúc 18:40

Bạn ra đề rất tùy tiện và không chịu check lại BĐT trước khi đăng:(

BĐT trên sai với [a = -1, b = -2, c = 3] thì Vế trái - Vế phải = -9/2 < 0.

Bình luận (0)
 Khách vãng lai đã xóa
tth_new
Xem chi tiết

ok , cảm ơn bạn !!!

Bài toán rất hay và bổ ích !!!

Bình luận (0)
Khôi Bùi
8 tháng 2 2019 lúc 20:21

Đây nhé 

Đặt b + c = x ; c + a = y ;  a + b = z 

\(\Rightarrow\hept{\begin{cases}x+y=2c+b+a=2c+z\\y+z=2a+b+c=2a+x\\x+z=2b+a+c=2b+y\end{cases}}\)

\(\Rightarrow\frac{x+y-z}{2}=c;\frac{y+z-x}{2}=a;\frac{x+z-y}{2}=b\)

Thay vào PT đã cho ở đề bài , ta có : 

\(\frac{y+z-x}{2x}+\frac{x+z-y}{2y}+\frac{x+y-z}{2z}\)

\(=\frac{1}{2}\left(\frac{y}{x}+\frac{z}{x}+\frac{x}{y}+\frac{z}{y}+\frac{x}{z}+\frac{y}{z}-3\right)\)

\(\ge\frac{1}{2}\left(2+2+2-3\right)=\frac{3}{2}\)

( cái này cô - si cho x/y + /x ; x/z + z/x ; y/z + z/y) 

Bình luận (0)
Phan Nghĩa
23 tháng 8 2020 lúc 21:22

e cũng có 1 vài cách chứng minh khá là cổ điển ạ !

Sử dụng BĐT AM-GM ta có :

\(\frac{a^2}{b+c}+\frac{b+c}{4}\ge2\sqrt{\frac{a^2}{b+c}.\frac{b+c}{4}}=2.\frac{a}{2}=a\)

Bằng cách chứng minh tương tự :

\(\frac{b^2}{a+c}+\frac{a+c}{4}\ge b;\frac{c^2}{a+b}+\frac{a+b}{4}\ge c\)

Cộng theo vế các bđt cùng chiều ta được :

\(\frac{a^2}{c+b}+\frac{b^2}{a+c}+\frac{c^2}{a+b}+\frac{2\left(a+b+c\right)}{4}\ge a+b+c\)

\(< =>\frac{a^2}{b+c}+\frac{a}{2}+\frac{b^2}{a+c}+\frac{b}{2}+\frac{c^2}{a+b}+\frac{c}{2}\ge a+b+c\)

\(< =>\frac{a^2}{b+c}+a+\frac{b^2}{a+c}+b+\frac{c^2}{a+b}+c\ge\frac{3}{2}\left(a+b+c\right)\)

\(< =>\frac{a\left(a+b+c\right)}{b+c}+\frac{b\left(a+b+c\right)}{a+c}+\frac{c\left(a+b+c\right)}{b+a}\ge\frac{3}{2}\left(a+b+c\right)\)

\(< =>\frac{a}{b+c}+\frac{b}{a+c}+\frac{c}{a+b}\ge\frac{3}{2}\left(Q.E.D\right)\)

Bình luận (0)
 Khách vãng lai đã xóa
Nguyễn Minh Đăng
Xem chi tiết
Xyz OLM
7 tháng 6 2021 lúc 17:02

\(\frac{a}{b+c}+\frac{b}{c+a}+\frac{c}{a+b}\ge\frac{3}{2}\)

<=> \(\frac{a+b+c}{b+c}+\frac{a+b+c}{c+a}+\frac{a+b+c}{a+b}\ge\frac{9}{2}\)

<=> \(2\left(a+b+c\right)\left(\frac{1}{b+c}+\frac{1}{c+a}+\frac{1}{a+b}\right)\ge9\)

<=> \(\left(a+b+b+c+c+a\right)\left(\frac{1}{b+c}+\frac{1}{c+a}+\frac{1}{a+b}\right)\ge9\)

<=> \(\frac{a+b}{b+c}+\frac{a+b}{c+a}+1+1+\frac{b+c}{c+a}+\frac{b+c}{a+b}+\frac{c+a}{b+c}+1+\frac{c+a}{a+b}\ge9\)

<=> \(\left(\frac{a+b}{b+c}+\frac{b+c}{a+b}\right)+\left(\frac{a+b}{c+a}+\frac{c+a}{a+b}\right)+\left(\frac{b+c}{c+a}+\frac{c+a}{b+c}\right)\ge6\)(đúng)

=> ĐPCM

Bình luận (0)
 Khách vãng lai đã xóa
Đanh Fuck Boy :))
7 tháng 6 2021 lúc 17:03

Mình làm cách đơn giản nhất nhá :))

Ta có:

\(\frac{a}{b+c}+\frac{b}{c+a}+\frac{c}{a+b}+3=\left(\frac{a}{b+c}+1\right)+\left(\frac{b}{c+a}+1\right)+\left(\frac{c}{a+b}+1\right)\)

\(=\left(a+b+c\right)\left(\frac{1}{b+c}+\frac{1}{c+a}+\frac{1}{a+b}\right)\ge\frac{9\left(a+b+c\right)}{2\left(a+b+c\right)}=\frac{9}{2}\left(Cauchy-Schwarz\right)\)

Hay \(\frac{a}{b+c}+\frac{b}{c+a}+\frac{c}{a+b}+3\ge\frac{9}{2}\Leftrightarrow\frac{a}{b+c}+\frac{b}{c+a}+\frac{c}{a+b}\ge\frac{3}{2}\)

Bình luận (0)
 Khách vãng lai đã xóa
Phan Nghĩa
10 tháng 6 2021 lúc 16:43

C1 : Bất đẳng thức ban đầu tương đương với :

\(2a\left(a+b\right)\left(a+c\right)+2b\left(b+c\right)\left(b+a\right)+2c\left(c+a\right)\left(c+b\right)\ge3\left(a+b\right)\left(b+c\right)\left(c+a\right)\)

\(< =>2\left(a^3+b^3+c^3\right)\ge ab\left(a+b\right)+bc\left(b+c\right)+ca\left(c+a\right)\)

\(< =>\left(a+b\right)\left(a-b\right)^2+\left(b+c\right)\left(b-c\right)^2+\left(c+a\right)\left(c-a\right)^2\ge0\)(ĐPCM)

Bình luận (0)
 Khách vãng lai đã xóa
Nguyen Thi Bich Huong
Xem chi tiết
Upin & Ipin
Xem chi tiết
Trí Tiên亗
27 tháng 2 2020 lúc 11:50

:33 Phương pháp SOS e chưa học và đọc :)) E làm các pp khác nhá anh :33

Cách 1 :Đặt : \(A=\frac{a}{b+c}+\frac{b}{a+c}+\frac{c}{a+b}\)

\(\frac{a^2}{ab+ac}+\frac{b^2}{ab+bc}+\frac{c^2}{ac+bc}\)

\(\ge\frac{\left(a+b+c\right)^2}{2\left(ab+bc+ca\right)}\ge\frac{3\left(ab+bc+ca\right)}{2\left(ab+bc+ca\right)}=\frac{3}{2}\)

Cách 2 : ( Kĩ thuật điểm rơi ) : Cộng 3 vào hai vế của BĐT rồi sử dụng AM - GM

Cách 3 : Nhân cả hai vế của BĐT với a+b+c

Cách 4 : Kĩ thuật đặt ẩn phụ ( Đặt a+b=x, b+c=y,c+a=z )

Bình luận (0)
 Khách vãng lai đã xóa
Trí Tiên亗
27 tháng 2 2020 lúc 11:58

Dùng phương pháp SOS :

Ta có : \(\sum_{} \) \(\frac{a}{b+c}-\frac{3}{2}\)\(\sum_{} \)\(\frac{\left(a-b\right)^2}{2\left(a+c\right)\left(b+c\right)}\ge0\) (1)

Vì a,b,c dương nên BĐT (1) đúng.

Dấu "=" xảy ra \(\Leftrightarrow a=b=c\)

Bình luận (0)
 Khách vãng lai đã xóa
tth_new
2 tháng 3 2020 lúc 13:14

Cách của bạn Đạt mà một kiểu SOS!

Đây là một kiểu khác (Của tthnew:v) S.O.C - Kĩ thuật phân tích bình phương cho bdt hoán vị - Tài liệu, chuyên đề, phương pháp về Bất đẳng thức - Diễn đàn Toán học. Và dưới đây là một kiểu khác:

\(VT-VP=\frac{1}{4\left(a+b+c\right)}\left[\Sigma_{cyc}\frac{\left(b+c-2a\right)^2}{b+c}\right]\ge0\)

Bình luận (0)
 Khách vãng lai đã xóa
M
Xem chi tiết
Phan Nghĩa
1 tháng 9 2020 lúc 15:27

Giả sử  \(\frac{a}{b}+\frac{b}{c}+\frac{c}{a}\ge\frac{a}{b+c}+\frac{b}{a+c}+\frac{c}{a+b}+\frac{3}{2}\)

\(< =>\frac{a}{b}+\frac{b}{c}+\frac{c}{a}\ge\frac{3}{2}+\frac{3}{2}=\frac{6}{2}=3\)(bđt nesbitt)

Giờ ta chỉ cần chỉ ra được \(\frac{a}{b}+\frac{b}{c}+\frac{c}{a}\ge3\) thì bài toán được hoàn tất chứng minh  

Thật vậy , theo BĐT Cauchy ta có \(\frac{a}{b}+\frac{b}{c}+\frac{c}{a}\ge3\sqrt[3]{\frac{a}{b}.\frac{b}{c}.\frac{c}{a}}=3\sqrt[3]{\frac{abc}{abc}}=3\)

Dấu "=" xảy ra khi và chỉ khi \(a=b=c\)

Vậy bài toán đã được hoàn tất chứng minh 

p/s : tí mình sẽ chứng minh bđt nesbitt ở dưới nhé

Bình luận (0)
 Khách vãng lai đã xóa
FL.Hermit
1 tháng 9 2020 lúc 15:29

BĐT cần CM <=>    \(\frac{a}{b}-\frac{a}{b+c}+\frac{b}{c}-\frac{b}{c+a}+\frac{c}{a}-\frac{c}{a+b}\ge\frac{3}{2}\)

<=>   \(\frac{ac}{b\left(b+c\right)}+\frac{ab}{c\left(c+a\right)}+\frac{bc}{a\left(a+b\right)}\ge\frac{3}{2}\)       (1)

Đặt:   \(A=\frac{ab}{c\left(c+a\right)}+\frac{bc}{a\left(a+b\right)}+\frac{ca}{b\left(b+c\right)}\)

\(A=\frac{a^2b^2}{abc\left(c+a\right)}+\frac{b^2c^2}{abc\left(a+b\right)}+\frac{c^2a^2}{abc\left(b+c\right)}\)

ÁP DỤNG BĐT CAUCHY - SCHWARZ SẼ ĐƯỢC:   

=>    \(A\ge\frac{\left(ab+bc+ca\right)^2}{abc\left(a+b+b+c+c+a\right)}=\frac{\left(ab+bc+ca\right)^2}{2abc\left(a+b+c\right)}\)    

TA TIẾP TỤC 1 BĐT:    \(\left(ab+bc+ca\right)^2\ge3abc\left(a+b+c\right)\)

=>    \(A\ge\frac{3abc\left(a+b+c\right)}{2abc\left(a+b+c\right)}=\frac{3}{2}\)         (2)

TỪ (1) VÀ (2) => TA CÓ ĐPCM.

Bình luận (0)
 Khách vãng lai đã xóa
FL.Hermit
1 tháng 9 2020 lúc 15:32

Arcobale_new làm kiểu gì thế ??

BĐT ngược chiều rồi nha bạn.

Bình luận (0)
 Khách vãng lai đã xóa
Hiền Hương
Xem chi tiết
Trần Thanh Phương
3 tháng 10 2019 lúc 18:12

Cách 1:

Áp dụng bđt Bunhiacopxki :

\(VT=\frac{a^2}{b+c}+\frac{b^2}{c+a}+\frac{c^2}{a+b}\ge\frac{\left(a+b+c\right)^2}{2\cdot\left(a+b+c\right)}=\frac{a+b+c}{2}\)

Dấu "=" xảy ra \(\Leftrightarrow a=b=c\)

Cách 2:

Áp dụng bđt Cô-si :

\(\frac{a^2}{b+c}+\frac{b+c}{4}\ge2\sqrt{\frac{a^2\cdot\left(b+c\right)}{4\cdot\left(b+c\right)}}=a\)

Tương tự : \(\frac{b^2}{c+a}+\frac{c+a}{4}\ge b\); \(\frac{c^2}{a+b}+\frac{a+b}{4}\ge c\)

Cộng vế :

\(\frac{a^2}{b+c}+\frac{b^2}{c+a}+\frac{c^2}{a+b}\ge a+b+c-\frac{2\left(a+b+c\right)}{4}=\frac{a+b+c}{2}\)

Dấu "=" xảy ra \(\Leftrightarrow a=b=c\)

Bình luận (4)
tthnew
23 tháng 11 2019 lúc 9:18

Cách 1: Svac:

\(VT\ge\frac{\left(a+b+c\right)^2}{2\left(a+b+c\right)}=\frac{a+b+c}{2}\)

Đẳng thức xảy ra khi a = b = c

Cách 2: SOS:

\(VT-VP=\left(\frac{a^2}{b+c}-\frac{a}{2}\right)+\left(\frac{b^2}{c+a}-\frac{b}{2}\right)+\left(\frac{c^2}{a+b}-\frac{c}{2}\right)\)

\(=\Sigma_{cyc}\left(\frac{a\left(a-b\right)}{2\left(b+c\right)}-\frac{b\left(a-b\right)}{2\left(c+a\right)}\right)=\Sigma\frac{\left(a-b\right)^2\left(a+b+c\right)}{2\left(b+c\right)\left(c+a\right)}\ge0\)

Vậy có đpcm.

Cách 3: Đợi tí em show hàng phương pháp mới:D

Bình luận (0)
 Khách vãng lai đã xóa
tthnew
24 tháng 11 2019 lúc 13:37

Giả sử \(c=min\left\{a,b,c\right\}\)

\(VT-VP=\frac{\left(a-b\right)^2\left(a+b+c\right)\left(7a+7b-2c\right)+\left(a+b-2c\right)^2\left(a+b+c\right)\left(a+b+2c\right)}{8\left(a+b\right)\left(a+c\right)\left(b+c\right)}\ge0\)

Bình luận (0)
 Khách vãng lai đã xóa
Nguyễn Thị Minh Nguyệt
Xem chi tiết
Nguyễn Thiều Công Thành
13 tháng 9 2017 lúc 22:39

áp dụng bđt cauchy ta có:

\(\frac{a^3}{b}+ab\ge2a^2;\frac{b^3}{c}+bc\ge2b^2;\frac{c^3}{a}+ca\ge2c^2\)

\(a^2+b^2+c^2\ge ab+bc+ca\)

\(\Rightarrow\frac{a^3}{b}+\frac{b^3}{c}+\frac{c^3}{a}\ge2\left(a^2+b^2+c^2\right)-ab-bc-ca\ge2\left(a^2+b^2+c^2\right)-a^2-b^2-c^2\)

\(=a^2+b^2+c^2\left(Q.E.D\right)\)

Bình luận (0)
Đinh Đức Hùng
13 tháng 9 2017 lúc 21:29

Theo Cauchy - Schwarz ta có : \(\left(a^2+b^2+c^2\right)\left(c^2+a^2+b^2\right)\ge\left(ab+bc+ac\right)^2\)

\(\Rightarrow a^2+b^2+c^2\ge\left|ab+bc+ac\right|\ge ab+ac+bc\)

Ta có : \(\frac{a^3}{b}+\frac{b^3}{c}+\frac{c^3}{a}=\frac{a^4}{ab}+\frac{b^4}{bc}+\frac{c^4}{ac}\ge\frac{\left(a^2+b^2+c^2\right)^2}{ab+ac+bc}\ge\frac{\left(a^2+b^2+c^2\right)^2}{a^2+b^2+c^2}\)

\(=a^2+b^2+c^2\)(đpcm)

Bình luận (0)